If Q is more popular than L, then each of the following must be true of the ranking EXCEPT:

jamelendez on April 30, 2020

Understanding my incorrect answer

On #11, after going through the problem, I was lead to believe that the correct answer was C. After reviewing the question and listening to the explanation as to why the answer is B, I'm still having trouble understanding why my answer was incorrect. May I have further clarification as to why choice C is wrong?

Replies
Create a free account to read and take part in forum discussions.

Already have an account? log in

Skylar on May 2, 2020

@Joslyn, happy to help!

We should combine the chain from our original game setup with the specific requirement in this question that Q > L to get the following sequence: 
                      > S
H > J > Q > L > V
                 > P     

Now, let's go through our answer choices and eliminate those that must always be true.

(A) "H is first" must always be true, so it is eliminated. We see that H is the only variable that does not have anything that must precede it. In turn, it must precede the other six variables. So, H must be first.

(B) "L is fourth" is not necessarily always true, so it is the correct answer. From the sequence we designed above, our chain could start with either "H > J > Q > L" or with "H > J > Q > P." So, it is possible that P is fourth instead of L, which means that L is not always fourth. 

(C) "V is not fourth" must always be true, so it is eliminated. As we established in the previous answer choice, the only two options for the fourth spot are L or P. Moreover, four variables need to precede V. Therefore, V will never be fourth.

(D) "J is not third" must always be true, so it is eliminated. Q will always be third, and J will always be second. This is because J is preceded only by H and must precede the remaining five variables.
 
(E) "Q is third" must always be true, so it is eliminated. As discussed above, Q is always third. It must be preceded by two variables (H and J) and must precede the remaining four variables. Therefore, it can only go third.

Does that make sense? Please let us know if you have any other questions!

jamelendez on May 29, 2020

Makes sense! Thank you!!

lgriffon22069 on July 20, 2020

Can I get an explanation on question 2 and 11 please? I can't seem to understand how the answer is the answer.